University Calculus: Early Transcendentals (3rd Edition)

Published by Pearson
ISBN 10: 0321999584
ISBN 13: 978-0-32199-958-0

Chapter 8 - Section 8.7 - Improper Integrals - Exercises - Page 471: 66

Answer

Diverges; $0$

Work Step by Step

Apply the limit comparison test. $\int_{0}^{\infty} \dfrac{ 2x dx}{x^2+1}=\lim\limits_{a \to \infty} \int_{0}^{a}\dfrac{2 x dx}{x^2+1} $ $\implies \lim\limits_{a \to \infty}[\ln (x^2+1)]_{-a}^a=\lim\limits_{a \to \infty} [\ln (a^2+1) -\ln 1]$ So, the integral $\int_{0}^{\infty} \dfrac{ 2x dx}{x^2+1}$ diverges and so, the integral $\int_{-\infty}^{\infty} \dfrac{ 2x dx}{x^2+1}$ also diverges. Now, $\lim\limits_{a \to \infty} \int_{-a}^a \dfrac{2 x dx}{x^2+1} \\=\lim\limits_{a \to \infty}[\ln (x^2+1)]_{-a}^a \\=\lim\limits_{a \to \infty} [\ln (a^2+1) -\ln (a^2+1)] \\=\lim\limits_{a \to \infty} (0) \\=0$
Update this answer!

You can help us out by revising, improving and updating this answer.

Update this answer

After you claim an answer you’ll have 24 hours to send in a draft. An editor will review the submission and either publish your submission or provide feedback.